Free DI and LR Practice Test - CAT 

Question 1

If the expenditure of Company Y in 1997 was Rs. 220 crores, what was its income in 1997?___

SOLUTION

Solution :

Profit percent of CompanyY in 1997 = 35.
Let the income of CompanyY in 1997 be Rs. x crores.
Then, 35 = x20220 × 100 x = 297.
Income of CompanyY in 1997 = Rs. 297 crores

Question 2

If X's income in 1999 and Y's expenditure in 2000 were equal , the income of X in 1999 and that of Y in 2000 are in the ratio :

A.

1:1

B.

2:3

C.

3:2

D.

Cannot be determined

SOLUTION

Solution : B

Let X's expenditure in 1999 be 100.
We have profit = 50%= ( Income - 100)100
Income = 150
As per the given information, we have Y's expenditure in 2000 = 150
So, again, ( Income - 150)150 = 50%
Income = 225

So, the required ratio is 150 : 225 = 2 : 3.

Question 3

In the bar graph shown here, the ratio of 'Exchange-traded funds (Commodity)' to 'Exchange-traded funds (Fixed Income)' was highest for which of the following years?

?

A.

2008

B.

2009

C.

2010

D.

2011 

SOLUTION

Solution : B

Just by observation, you can be sure that 2008 can be eliminated. Now among 2010 and 2011, the 'Equity' part is same for both of them. To have a higher ratio value, the 'commodity' part should be higher and 'Fixed Income' part should be lower. Thus among these two years, we can again eliminate 2011. The following table will give you the approximate value for years 2009 & 2010.

YearCommodityFixedIncomeRatio20090.040.200.2020100.020.160.125

Question 4

Which of the following statements is necessarily true?

A. Total population of South Indians in the four cities together is less than 4, 60,001.
B. Total population of North Indians in the four cities together is greater than 2, 80,000
C. Total population of West Indians in the four cities together is 4, 40,000
D. None of the above is necessarily true

SOLUTION

Solution : A

The maximum population of south India could be 23% of total population i.e. 4,60,000.

 

Question 5

What can be said regarding the two statements if it is known that number of people in each of the cities is an integral multiple of 100?

Statement 1: - The total number of people in Bangalore is less than that in Mumbai

Statement 2: - The total number of people from north India in Bangalore is more than the number of people from north India in Mumbai.

A. Both statements are true.
B. Both statements are false.
C. If statement 1 is true, then statement 2 is false.
D. If statement 1 is true, only then, statement 2 is true

SOLUTION

Solution : C

Statement 1- B<M
Statement 2- NB>NM=>13% of B> 15% of M (This is possible only if B>M)
Thus, if statement 1 is true, statement 2 is definitely false.

 

Question 6

If the consumption of each petroleum product in 2011 is higher than that in 2010 then the minimum growth in total consumption in 2011 is more than:

A.

20%

B.

14.28%

C.

16.66%

D.

Cannot be determined

SOLUTION

Solution : A

Here, Let total consumption of petroleum products in 2010 is x and in 2011 is y.
Now consider the component where the percentage change between the two years is maximum (this is Kerosene)
To compute minimum growth, 12x = 10y, so x = 10 and y = 12,

Growth = yxx = 20%.

Question 7

If Kerosene consumption in 2011 is 40% higher than kerosene consumption in 2010. Then the ratio of petrol in 2011 to 2010 (Approx):

A.

147:100

B.

8:7

C.

7:5

D.

Cannot be determined

SOLUTION

Solution : A

If kerosene consumption in 2010 is 100 then in 2011 it should be 140. The total consumption in 2010 will be 25003  and in 2011 is 1400. Then the ratio of petrol in 2011 to 2010 is 147:100.

 

Question 8

What is the ratio of total consumption of petroleum products in 2010 to that in 2011?

A.

10:12 

B.

23:29

C.

1:1

D.

Cannot be determined

SOLUTION

Solution : D

We don't have any link between 2010 and 2011. So, we cannot calculate the ratio of total consumption of petroleum products in 2010 to that in 2011.

Question 9

What is the difference in the revenues (in crore) of the company for the two years 2008 and 2009, if the revenue of the company in the year 2009 was 432 crore.___

SOLUTION

Solution : Revenue in the year 2008 = (4321.08)= 400,

Difference in revenue = 432 - 400 = 32 crore.

 

 

 

Question 10

Approximately what percentage of the revenue in the year 2011, was the revenue in the year 2009?

A.

75%

B.

25%

C.

80%

D.

None of these

SOLUTION

Solution : C

Percentage increase in the revenue in 2011, as compared to 2009.
= 16 + 7.5 + 16×7.5100 = 23.5 + 1.2 = 24.7
Required percentage = 100124.7 × 100 = 80% (approximately)

Question 11

If the horses complete the race, from the first to the last, in exactly the order 6, 5, 4, 3, 2 and 1 (corresponding to the number of their jockeys); and if B is the horse that wins the race, then each of the following horses could have been among the top three in the race except:

A.

C

B.

D

C.

E

D.

F

SOLUTION

Solution : A

As per the question,the top 3 are ridden by jockeys 6,5 and 4.B is ridden by jockey-6.So from condition (iii), E has to be ridden by jockey-4.Among the top 3, we are only left with jockey 5.But jockey 5 cannot ride C as per condition(iv).Hence option (a)

Question 12

If Jockey 5 is the one not assigned to any horse, which of the following could be true?

A.

A is ridden by Jockey 4

B.

F is ridden by Jockey 6

C.

B is ridden by Jockey 6

D.

C is ridden by Jockey 7

SOLUTION

Solution : B

If Jockey 5 has not been assigned any horse,then E is ridden by jockey 4.Consequently,from condition(iii),B cannot be ridden by Jockey 6.As per condition(iv), we cannot have jockey 7 riding C.So options(a),(c) and (d) are eliminated.

Question 13

If D is incapable of running the race and no replacement horse is found; and if the horses that run, finish the race from the first to the last, in the order 1, 2, 4, 6 and 7 (corresponding to the numbers of their jockeys) then which of the following must have finished last in the race?

A.

A

B.

B

C.

C

D.

E

SOLUTION

Solution : B

As per the question, jockey 5 has not been assigned any horse and hence E has to be assigned to jockey 4 . So B cannot be assigned to jockeys 2 and 6 as per condition(iii).Also, A or C should be ridden by jockey 1 which leaves us with only one choice wherein B is ridden by jockey 7.Hence option(b).

Question 14

Farida can review any of the following except

A.

Kahaani with Bindu

B.

Kahaani with Esha

C.

Paansinghtomar with Esha

D.

Agent vinod with Esha

SOLUTION

Solution : B

Chandan is not reviewing Housefull 2. So, from given conditions Anil will not review Housefull 2 
KahaaniPaansinghtomarAgentVinodHousefull2AnilXBinduXXXChandanXDeepakXXEshaFarida

From the given table it is clear, at least one between Esha and Farida will review Housefull 2.
From the statement Farida reviews the same movie as exactly one other reviewer, we can easily determine option (b) is not possible because Bindu is reviewing Kahaani.

Question 15

Which one of the following can be true?

A.

Anil and Bindu review Kahaani

B.

Anil and Farida review Housefull 2

C.

Bindu and Deepak review Kahaani

D.

Farida reviews Paansinghtomar and Anil reviews Agent vinod

SOLUTION

Solution : D

Option(a) Anil and Bindu review Kahaani. It means Chandan will review Kahaani. From the given condition "F reviews the same movie as exactly one other reviewer”. So, Option (a) is not possible
Option(b) Anil and Farida review Housefull 2. From, the given table, It is clear that A cannot review Housefull 2. Option (b) is not possible
Option(c) Bindu and Deepak review Kahaani. From the given conditions: - . "C reviews the same movie as A and F reviews the same movie as exactly one other reviewer” we cannot fulfill the condition that all 4 movies are reviewed. Option (c) is not possible

Option(d) is the right answer.

Question 16

Which one of the following, if substituted for the condition that motorbike servicing has to be earlier than laundry, would have the same effect in determining the order of the student's activities?

A.

Laundry has to be one of the last three activities.

B.

Laundry has to be either immediately before or immediately after jogging.

C.

Jogging has to be earlier than laundry.

D.

Laundry has to be earlier than hedge trimming.

SOLUTION

Solution : C

The fourth condition in the passage tells you that motorbike servicing has to be either immediately before or immediately after jogging. That is, M and J must be ordered as a block, either M J or JM, with respect to the other four activities. Thus, if, as the original third condition states, M has to be earlier than L, then we know that J must also be earlier than L. Conversely, if, as the new condition in answer choice (C) states, J has to be earlier than L, then we know that M must also be earlier than L. In short, the third condition and answer choice (C) have exactly the same effect. Therefore, answer choice (C) is correct.

Question 17

Which statement is definitely correct?

A.

Kitchen cleaning has to be earlier than jogging.

B.

Jogging has to be earlier than Grocery shopping.

C.

Motorbike servicing has to be earlier than Grocery shopping.

D.

Kitchen cleaning has to be earlier than hedge trimming.

SOLUTION

Solution : D

From the given conditions, we can easily determine "Kitchen cleaning has to be earlier than hedge trimming” Option(d).

Question 18

The ratio of the maximum possible score in the written test to the maximum possible score in the personal interview is x : 1. Find the value of 'x'
___

SOLUTION

Solution : We are given all the percentage scores of Bharat, with which we can use to find the maximum possible scores in the written test, GD and PI.
Let the maximum possible score in the written test be 'x'and the personal interview and GD be 'y'each.
x+ 2y= 500                                       ... (i)

Now, Bharat's percentage score in the written test is given by,

40+54+70+804 = 61%
 61x100+76y100+83y100 = 68.4 × 500100
61x + 159y = 34200                  ........(ii)
Multiplying (i) by 61 and subtracting from (ii), we get,
37y = 3700
y = 100
x = 300
x:y = 3:1

 

Question 19

How much did Amar score in the QA section?
___

SOLUTION

Solution : Based on the answer to the previous question, the maximum score in the written test is 300, and so the maximum score in each section is 75, while the maximum score in the GD and PI is 100 each.
Thus, we can now calculate the missing percentage scores in the given table.
Amar's total percentage is 66.3, so his total score is 331.5 out of 500.
Out of this, 75 + 81 = 156 out of 200 was scored in the GD and PI
He scored 175.5 out of 300 in the written test.

So, if Amar scoredxout of 75 in QA, then we have,

x + [(50+68+72)×75100] = 175.5
175.5 - 142.5 = 33

 

Question 20

If the five friends are ranked in descending order according to their total marks, Daya is ranked third and has one mark more than the person who ranked fourth. How much did Daya score in the GD?
___

SOLUTION

Solution :

According to the given conditions, Daya should have had one mark more than Amar.
 Daya got 332.5 out of 500 or 66.5%.
HIs score in the GD = 332.5 - 77 - [(58+78+76+42)×75100]
                                               = 255.5 - 190.5 
                                               = 65

Question 21

If the difference between Eesha's marks in DI and VA is equal to the difference between the total marks of Amar and Bharat, then what is the sum of Eesha's score in DI and VA?
___

SOLUTION

Solution :

The total marks of Amar and Bharat are 331.5 and 342 respectively, so the difference between them is 10.5.
Now, Eesha's total marks are 
68.8 × 500100 = 344
Out of this, she scored 79 in the GD, 85 in the PI and 
40 × 75100 = 30 in QA 
66 × 75100 = 49.5 in LR 
 Eesha scored 344 - 79 - 85 - 30 - 49.5 = 100.5 in (DI + VA) 

Question 22

If (P3,P4) share a flat, then the other flat P4 shares will be

A.

A

B.

B

C.

D

D.

E

SOLUTION

Solution : B

From the given passage, we can conclude that

A -- P1
B -- P2
C -- not P4
D -- not P1 and P4
E -- not P2

If P3 and P4 are sharing flat, then

A -- P1 and P5
B -- P2 and P4
C -- P1 and P3
D -- P5 and P2
E -- P3 and P4

Question 23

If (P4,P5) share a flat, then the other flat P4 shares will be

A.

A

B.

B

C.

D

D.

E

SOLUTION

Solution : A

From the given passage, we can conclude that

A -- P1
B -- P2
C -- not P4
D -- not P1 and P4
E -- not P2

If P3 and P4 are sharing flat, then

A -- P1 and P4
B -- P2 and P3
C -- P1 and P3
D -- P5 and P2
E -- P5 and P4

Question 24

Which fruit is third from the extreme left?

A.

Apple

B.

Mango

C.

Pineapple

D.

Banana

SOLUTION

Solution : C

Papaya, Orange, Pineapple, Mango, Banana, Watermelon, Apple

Question 25

Find the number of fruits between Mango and Watermelon.

A.

0

B.

1

C.

2

D.

3

SOLUTION

Solution : B

Papaya, Orange, Pineapple, Mango, Banana, Watermelon, Apple

Question 26

Which fruit is at the extreme right?

A.

Apple

B.

Banana

C.

Pineapple

D.

Either (a) or (b)

SOLUTION

Solution : A

Papaya, Orange, Pineapple, Mango, Banana, Watermelon, Apple

Question 27

Whenever the wind blows too hard, the doors and windows are not opened.
A. The doors and windows are opened.
B. The doors and windows are not opened.
C. The wind blew too hard.
D. The wind did not blow too hard.

A.

AC

B.

DA

C.

AD

D.

BC

SOLUTION

Solution : C

Whenever X occurs, Y occurs, however whenever X does not occur, one cannot say what is true of Y. Therefore AD is valid, but not DA.

Question 28

If the lisps are not corrected in time, the children will not be able to communicate properly.
A. The lisps are not corrected in time.
B. The lisps are corrected in time.
C. The children will not be able to communicate clearly.
D. The children will be able to communicate clearly.

A.

DB

B.

BC

C.

AD

D.

CB

SOLUTION

Solution : A

If X then Y, which means that if Y has not occurred, then X has not either.

Question 29

Four people need to cross a bridge at night. They have one flashlight for the four of them, but the bridge is only wide enough for two to cross at a time. Because it's so dark, anyone crossing the bridge must do so with the flashlight. Person A can walk across in 1 minute. Person B takes 2 minutes; person C takes 5 minutes, and finally person D needs a full 10 minutes to cross. Naturally, if two people are crossing the bridge, they move with the speed of the slower person. What is the shortest amount of time it will take for all four to get to the other side?

A.

18

B.

8

C.

17

D.

15

SOLUTION

Solution : C

The shortest time is 17 minutes. The key to this problem is that to minimize the time spent; we need to have persons C and D to cross together. However, to avoid one of them having to make the trip back, we need to ensure that when they cross, someone is waiting for them on the other side to take the flashlight back.

Question 30

What was the number of extras in the Indian score?

A.

10

SOLUTION

Solution : A

It is given that the first a batsmen scored multiples of 5. Also the first two scored a total of 50 and the ratio of scores of the first three batsmen Ganguly, Sehwag and Pathan was 6 : 4 : 3. i.e. either 6x + 4x = 50 or 6x + 3x = 50 or 4x + 3x = 50
As the only possibility is 6x + 4x = 50, Ganguly scored 30, Sehwag scored 20 and Pathan scored 15. Hence Pathan was 3rd in the batting order.
Yuvraj scored 5 times that of Sehwag, i.e. 100.
Tendulkar scored equal to Sehwag, i.e. 20
Srinath scored 5 times Tendulkar, i.e.100
Hence Yuvraj and Srinath occupy positions 4 and 5, not necessarily in that order
Ganguly = Tendulkar + Extras. Therefore, Extras = 10
Munaf and Mohanty are batsmen 10 and 11, though not necessarily in that order.
Together they score 40 which is equal to 25 + Dhoni + Pathan. Hence Dhoni = 0

Batsmen 9 scored 5. As the two who score 0 are consecutive, batsmen 7 scores 0. Hence Tendulkar has only one slot left, i.e. 6. Now all questions can be answered. 

Question 31

Tendulkar's number in the batting order was immediately after

A. Yuvraj
B. Srinath
C. Yuvaraj or Srinath
D. Cannot be determined

SOLUTION

Solution : C

It is given that the first a batsmen scored multiples of 5. Also the first two scored a total of 50 and the ratio of scores of the first three batsmen Ganguly, Sehwag and Pathan was 6 : 4 : 3. i.e. either 6x + 4x = 50 or 6x + 3x = 50 or 4x + 3x = 50
As the only possibility is 6x + 4x = 50, Ganguly scored 30, Sehwag scored 20 and Pathan scored 15. Hence Pathan was 3rd in the batting order.
Yuvraj scored 5 times that of Sehwag, i.e. 100.
Tendulkar scored equal to Sehwag, i.e. 20
Srinath scored 5 times Tendulkar, i.e.100
Hence Yuvraj and Srinath occupy positions 4 and 5, not necessarily in that order
Ganguly = Tendulkar + Extras. Therefore, Extras = 10
Munaf and Mohanty are batsmen 10 and 11, though not necessarily in that order.
Together they score 40 which is equal to 25 + Dhoni + Pathan. Hence Dhoni = 0

Batsmen 9 scored 5. As the two who score 0 are consecutive, batsmen 7 scores 0. Hence Tendulkar has only one slot left, i.e. 6. Now all questions can be answered. 

Question 32

What is the total of the least 5 scores in the Indian Cricket team (excluding extras)?___

SOLUTION

Solution : It is given that the first a batsmen scored multiples of 5. Also the first two scored a total of 50 and the ratio of scores of the first three batsmen Ganguly, Sehwag and Pathan was 6 : 4 : 3. i.e. either 6x + 4x = 50 or 6x + 3x = 50 or 4x + 3x = 50
As the only possibility is 6x + 4x = 50, Ganguly scored 30, Sehwag scored 20 and Pathan scored 15. Hence Pathan was 3rd in the batting order.
Yuvraj scored 5 times that of Sehwag, i.e. 100.
Tendulkar scored equal to Sehwag, i.e. 20
Srinath scored 5 times Tendulkar, i.e.100
Hence Yuvraj and Srinath occupy positions 4 and 5, not necessarily in that order
Ganguly = Tendulkar + Extras. Therefore, Extras = 10
Munaf and Mohanty are batsmen 10 and 11, though not necessarily in that order.
Together they score 40 which is equal to 25 + Dhoni + Pathan. Hence Dhoni = 0

Batsmen 9 scored 5. As the two who score 0 are consecutive, batsmen 7 scores 0. Hence Tendulkar has only one slot left, i.e. 6.